GMAT Critical Reasoning Assumption Questions Practice

The Assumption Questions in GMAT's Critical Reasoning section tell you what the conclusion is and then ask you to go back in the argument and see what must have been true for the argument to be true. Premise (P) + Assumption (A) = Conclusion (C). The tempting wrong answers in the answer choices are another conclusion statement. That is definitely a wrong answer. Below are some sample questions from the GMAC's paper Test Code 25 and various other sources including Manhattan Elite Prep Verbal Study Workbook for illustrative teaching purposes.

GMAT Critical Reasoning Assumption Practice Question 1

Bank depositors in the United States are all financially protected against bank failure because the government insures all individuals' bank deposits. An economist argues that this insurance is partly responsible for the high rate of bank failures, since it removes from depositors any financial incentive to find out whether the bank that holds their money is secure against failure. If depositors were more selective, then banks would need to be financially stable in order to compete for depositors' money.

The economist's argument makes which of the following assumptions?

(A) Bank failures are caused when big borrowers default on loan repayments.

(B) A significant proportion of depositors maintain accounts at several different banks.

(C) The more a depositor has to deposit, the more careful he or she tends to be in selecting a bank.

(D) The difference in the interest rates paid to depositors by different banks is not a significant factor in bank failures.

(E) Potential depositors are able to determine which banks are financially secure against failure.

Solutions: *This is an assumption question.

The correct answer is (E). It is a fairly straightforward question and the wrong answers are easy to eliminate.

GMAT Critical Reasoning Assumption Practice Question 1 - Part 2

Bank depositaries in the US are all financially protected against bank failure because the government insures all individual's bank deposits. An economist argues that this insurance is partly responsibly for the high rate of bank failures, since it removes from depositors any financial incentive to find out where the bank that holds heir money is secure against failure. If depositors were more selective, then banks would need to be financially secure in order to compete for depositor's money.

Which of the following, if true, most seriously weakens the economist's argument?

(A) Before the government started to insure depositors against bank failure, there was a lower rate of bank failure than there is now.

(B) When the government did not insure deposits, frequent banks failures occurred as result of depositor's fears or losing money in bank failures.

(C) Surveys show that a significant proportion of depositors are aware that their deposits are insured by the government.

(D) There is an upper limit on the amount of an individual's deposit that the government will insure, but very few individuals' deposits exceed this limit.

(E) The financial security of a bank against failure depends on the percentage of its assets that are loaned out and also on how much risk its loan involve.

Solutions: *(E) is a tempting answer because it suggests that insurance is not the main factor in the fight against bank failure. The correct answer is (B); when there were no deposits insured then there were no bank failures.

GMAT Critical Reasoning Assumption Practice Question 2

The Flerenchian government deicded to limit the import of chocolate from the four countries which export the greatest amount of chocolate to Flerenchia. An analyst hired by the government maintains that in the near future this will cause a large increase in domestic sales of chocolate produced in Flerenchia.

Which of the following, if true, would most likely renders this prediction inaccurate?

(A) A new tax bill that would discourage foreign investment in the chocolate industry is being debated by the Flerenchian government.

(B) Flerenchian companies' orders for milk chocolates, which account for 60% of sales by chocolate companies, rose faster than for other types of chocolates during the past year.

(C) Worldwide order for chocolate made in Flerenchia rose more than 15% during the past year.

(D) Substantial inventories of foreign-made chocolates were stockpiled in Flerenchia during the past year.

(E) Companies in the chocolate industries of many countries showed significantly increased demand for chocolate during the past year.

Solutions: *This falls into a special assumptions: predictions questions.

What the analyst is predicting is that there will be a large increase in domestic sales in the future. (B) is incorrect because it is out of the scope of the question. (A) is the most tempting wrong answer. There are two problems with (A): the tax bill is a 'maybe' and (A) talks about the medium term and we're looking to see what will happen over the long term. (D) is the correct answer. (E) is not true because it's a strengthener questions.

GMAT Critical Reasoning Assumption Practice Question 3

Because postage rates are rising, Home Decorator magazine plans to maximize its profits by reducing by one half the number of issues it publishes each year. The quality of articles, the number of articles published per year, and the subscription price will not change. Market research shows that neither subscribers nor advertisers will be lost if the magazine's plan is instituted.

Which of the following, if true, proved the strongest evidence that the magazine's profits are likely to decline if the plan is instituted?

(A) with the new postage rates, a typical issue under the proposed plan would cost about one-third more to mail than a typical current issue would.

(B) The majority of the magazine's subscribers are less concerned about a possible reduction in the quantity of the magazine's articles than about a possible loss of the current high quality of it's articles.

(C) Many of them magazine's long-time subscribers would continue their subscriptions even if the subscription price were increase.

(D) Most of the advertisers that purchase advertising space in the magazine will continue to spend the same amount on advertising per issue as they have in the past.

(E) Production costs for the magazine are expected to remain stable.

Solutions: *This is a weakening question. It talks about a plan and we are looking for an answer that will assume that the plan will not work. (A) is not correct because it will not make the magazine's profits decline. (C) is a strengthener answer. The correct answer is (D) because it will cause the profits to decline.

GMAT Critical Reasoning Assumption Practice Question 4

A study of marital relationships in which one partner's sleeping and waking cycles differ from those of the other partner reveals that such couples share fewer activities with each other and have more violent arguments than do couples in a relationship in which both partners follow the same sleeping and waking patterns. Thus, mismatched sleeping and waking cycles can seriously jeopardize a marriage.

Which of the following, if true, most seriously weakens the argument above?

(A) married couples in which both spouses follow the same sleeping and waking patterns also occasionally have arguments that can jeopardize the couple's marriage.

(B) The sleeping and waking cycles of individuals tend to vary from season to season.

(C) The individuals who have sleeping and waking cycles that differ significantly from those of their spouses tend to argue little with colleagues at work.

(D) People in unhappy marriages have been found to express hostility by adopting a different sleeping and waking cycle that from that of their spouses.

(E) According to a recent study,. Most people's sleeping and waking cycles can be controlled and modified easily.

Solutions: *We are looking for a weakener to the argument. The correct answer is (D) since it argues that the reasons for having different sleeping and waking cycles for a couple could be their unhappiness to begin with, which is the opposite of what the statement in the questions.